0 Daumen
300 Aufrufe

Aufgabe

Wir wissen, dass für alle \( h \in[0,1] \) :
\( 1+h \leq \exp (h) \leq 1+2 h . \)

Ich muss den Logarithmus auf die obige Ungleichung anwenden und daraus schliessen, dass für alle \( k, n \in \mathbb{N} \) mit \( 1 \leq k \leq n-1 \) :
\( \frac{1}{2(n-k)} \leq \log \left(\frac{n-k+1}{n-k}\right) \leq \frac{1}{n-k} . \)


Problem/Ansatz:

Könnte jemand mir gerne helfen? Danke

Avatar von

2 Antworten

0 Daumen
 
Beste Antwort

Den ersten Teil vielleicht so:

Seien \( k, n \in \mathbb{N} \) mit \( 1 \leq k \leq n-1 \) .

==>    \( 1 \leq k \leq n-1 \)

==>    \( 2 \leq k+1 \leq n \)

==>    \( 2-k \leq 1 \leq n-k \) 

Insbesondere also    \(  1 \leq n-k \) und dann gilt für den Kehrwert

\(  0 \leq \frac{1}{n-k} \leq 1 \) und ist damit geeignet als das h in

der gegebenen Ungleichung:

\(1+ \frac{1}{n-k} \leq \exp ( \frac{1}{n-k}) \leq 1+ \frac{2}{n-k} \)

\(\frac{n-k+1}{n-k} \leq \exp ( \frac{1}{n-k}) \leq \frac{n-k+2}{n-k} \)

Alles ist positiv und ln streng monoton steigend, also

\(     ln(\frac{n-k+1}{n-k} )\leq \frac{1}{n-k} \leq  ln(\frac{n-k+2}{n-k} )\)

Damit hast du zumindest schon mal den 2. Teil der

zu beweisenden Ungleichung.

Avatar von 288 k 🚀

danke

und wie kann ich aus dieser Aufgabe herleiten, am einfachsten, dass

für alle \( n \in \mathbb{N} \) mit \( n \geq 1 \) her:
\( \frac{1}{2} \sum \limits_{k=1}^{n-1} \frac{1}{k} \leq \log (n) \leq \sum \limits_{k=1}^{n-1} \frac{1}{k} \)  ist. Danke

0 Daumen

Aus der Ungleichung für \(\exp(h)\) folgt wegen der Monotonie von \(\ln\):

\(\ln(1+h)\leq h\) und \(h \leq \ln(1+2h)\).

In die erste Ungleichung setze man \(h=\frac{1}{n-k}\) ein:

\(\ln(\frac{n-k+1}{n-k})\leq \frac{1}{n-k}\).

In die zweite Ungleichung setze man \(h=\frac{1}{2(n-k)}\) ein:

\(\frac{1}{2(n-k)}\leq \ln(\frac{n-k+1}{n-k})\).

Avatar von 29 k

Ein anderes Problem?

Stell deine Frage

Willkommen bei der Mathelounge! Stell deine Frage einfach und kostenlos

x
Made by a lovely community